Inégalité avec une intégrale

Bonjour

$f : [0;1] \to \R$ de classe $C^3$, telle que $ f(0)=f’(0)=f(1)=0$ et $|f’’’| \leq 1.$
Montrer que $$
\sqrt{3}\,|f(x)| \leq x(1-x)\sqrt{\int_{0}^{x} \frac{|f(t)|}{t(1-t)} dt}.

$$ Merci.

Réponses

  • Bonjour

    J’ai cherché. Je n’ai pas trouvé.

    Et vous ?
  • supp
  • J'ai trouvé une autre façon de prouver $|g'|\leq \dfrac{1}{6}$. Je reprends les notations de side.

    Tout d'abord, on remarque que pour tout $x\in \left]0,1\right[$ :
    \[\begin{align} g(x)&=\frac{f(x)}{x(1-x)}\\
    &=\frac{f(x)-f(0)}{x-0}-\frac{f(1)-f(x)}{1-x}\\
    &=\frac{1}{x}\int_0^x f'(u)du - \frac{1}{1-x}\int_x^1 f'(v)dv\\
    &=\int_0^1 f'(tx)dt - \int_0^1 f'((1-t)x+t)dt\\
    &=\int_0^1 \left(f'(tx)-f'((1-t)x+t)\right)dt
    \end{align}\]

    La formule est encore valable pour $x=0$ et $x=1$.

    On peut alors dériver sous le signe intégral pour obtenir ensuite par intégration par parties pour tout $x\in [0,1]$ :
    \[\begin{align}g'(x)&=\int_0^1 \left(tf''(tx)-(1-t)f''((1-t)x+t)\right)dt\\
    &=\left[\frac{t^2}{2}f''(tx)+\frac{(1-t)^2}{2}f''((1-t)x+t)\right]_0^1-\int_0^1\left(x\frac{t^2}{2}f^{(3)}(tx)+(1-x)\frac{(1-t)^2}{2}f^{(3)}((1-t)x+t)\right)dt\\
    &=0-\frac{1}{2}\int_0^1\left(xt^2f^{(3)}(tx)+(1-x)(1-t)^2f^{(3)}((1-t)x+t)\right)dt\end{align}\]

    Par conséquent, en utilisant l'inégalité triangulaire et $|f^{(3)}|\leq 1$, on obtient directement pour tout $x\in [0,1]$ :
    \[ |g'(x)|\leq \frac{1}{2}\left(x\int_0^1 t^2dt+(1-x)\int_0^1 (1-t)^2dt\right)=\frac{x+(1-x)}{6}=\frac{1}{6}\]

    On peut alors conclure comme side l'a fait, pour tout $x\in [0,1]$ \[F'(x)=|g(x)|-6g(x)g'(x)\geq |g(x)|-6|g(x)| |g'(x)|\geq 0\]
    Donc $F$ est croissante sur $[0,1]$ et nulle en 0 donc elle est positive, ce qui termine l'exercice.
  • Bonjour,

    Je suis admiratif. (tu)
  • Il y a égalité pour $f_1(x)=\dfrac{x^2(1-x)}6$ et $f_2=-f_1$.
  • @YvesM , side , bisam, jandri merci .

    Il y a une autre noisette à décortiquer
    http://www.les-mathematiques.net/phorum/read.php?4,2009524
  • J'ai repris l'astuce de bisam (à laquelle je n'aurais pas pensé tout seul) mais je n'ai pas fait exactement le même changement de variable pour la deuxième intégrale : au lieu de poser $v=(1-t)x+t$ j'ai posé $v=1+(x-1)t$.
    Cela donne $g(x)=\displaystyle\int_0^1 \left(f'(tx)-f'(1+(x-1)t)\right)dt$ d'où $g'(x)=\displaystyle\int_0^1t \left(f''(tx)-f''(1+(x-1)t)\right)dt$.
    L'inégalité des accroissements finis donne alors directement $|g'(x)|\leq \displaystyle\int_0^1t(1-t)||f^{(3)}||_{\infty}dt\leq \dfrac16$.
Connectez-vous ou Inscrivez-vous pour répondre.